Search found 53 matches


IMO C C)Will having judges give instructions at the beginning of a trial rather than at the end significantly alter the customary procedures employed by the judicial system? If judge’s instructions at the beginning of a trial rather than at the end permit jurors to concentrate on the most relevant...

by agarwalmanoj2000

Sun Apr 01, 2012 10:25 pm
Forum: Critical Reasoning
Topic: Kaptest- Experts have a shot at this please
Replies: 4
Views: 1073

Meadrow – 1000 people; 4 year ago violent crime rate 10%; Current violent crime rate 16%. Parkdale – 1000 people; 4 year ago violent crime rate 50%; Current violent crime rate 55%. Thus residents of Meadowbrook are NOT more likely to become victims of violent crime than are residents of Parkdale...

by agarwalmanoj2000

Sun Apr 01, 2012 8:25 pm
Forum: Critical Reasoning
Topic: FLAW.. interesting???
Replies: 5
Views: 1688

Hi Bill, Very nice explanation for option E. Please also explain why not C? K – Let us say avg. per capita income $12K. 50 people live under poverty out of total population of 2000. B - Let us say avg. per capita income $15K. 50 people live under poverty out of total population of 100. This also m...

by agarwalmanoj2000

Sun Apr 01, 2012 8:04 pm
Forum: Critical Reasoning
Topic: Weaken Problem.. Please help...
Replies: 2
Views: 965

Conclusion: Yoga teachers should choose to focus primarily on exercises that strengthen muscles. Goal: Find assumption. On negating the correct answer choice conclusion should fall apart. Why not B? Negate B: Different teachers are NOT able to interpret yoga poses in different ways. =>It does not im...

by agarwalmanoj2000

Sun Apr 01, 2012 7:18 pm
Forum: Critical Reasoning
Topic: Yoga can help
Replies: 4
Views: 1219

IMO E We need to find and option that says verbal cursing behavior DOES NOT originates in the same speech centers as other types of speech Only option E relates to verbal cursing behavior origin. It says although speech centers is damaged, verbal cursing continues under stress, so verbal cursing mus...

by agarwalmanoj2000

Thu Mar 29, 2012 8:19 pm
Forum: Critical Reasoning
Topic: Kaplan GMAT
Replies: 5
Views: 1497

Dear Mitch, Thanks a lot for your great response. I understand option E is wrong because it contradicts the premises of the argument. Can I generalize this and cancel the answer choices, if they contradict the premises of the argument in assumption, strengthen and weaken type CR question? Please gui...

by agarwalmanoj2000

Thu Mar 29, 2012 7:59 pm
Forum: Critical Reasoning
Topic: Computer-based activities
Replies: 8
Views: 6382

1) IMO A –
Bassed on last sentence of 2nd para -
When the right and means of absolute command are conferred on a people or a king, on an aristocracy or a democracy, a monarchy or a republic, there has been implanted the germ of tyranny.

2) I could not get the answer so guessed A.

3) IMO C

by agarwalmanoj2000

Wed Mar 28, 2012 8:18 pm
Forum: Reading Comprehension
Topic: Majority Tyranny
Replies: 5
Views: 2088

Conclusion: The number of disabled people who are involuntarily unemployed will drop substantially. (A) Many congressmen were reluctant to pass the new legislation to prevent discrimination against the disabled. => Out of scope. Congressmen's intention is not related to conclusion. (B) Some private ...

by agarwalmanoj2000

Wed Mar 28, 2012 6:09 pm
Forum: Critical Reasoning
Topic: law for federal
Replies: 3
Views: 1231

(C) The immature woks by Renoir and Cezanne that were purchased by Ms. Federici were at that time thought by some critics to be unimportant juvenile works. Option C says - Critics said works by Renoir and Cezanne that were purchased were not good. This strenthens the argument by reinforcing that mus...

by agarwalmanoj2000

Wed Mar 28, 2012 5:55 pm
Forum: Critical Reasoning
Topic: MUSEUM
Replies: 3
Views: 1374

Strengthen evidence was inadmissible. (A) Flight from the police could create a reasonable suspicion of a criminal act as long as other significant factors are involved. => Weakens as it justifies flight could create a reasonable suspicion. (B) People can legally flee from the police only when those...

by agarwalmanoj2000

Wed Mar 28, 2012 5:47 pm
Forum: Critical Reasoning
Topic: POLICE CHASE
Replies: 3
Views: 1220

in C also they are not continuing to pay the money to repair the pipe Yes as per option C taxpayers are not paying the money to repair the pipe, but we have to find a reason for taxpayer for not providing funds to train workers. Tax payers cannot use the reason that city council has cut the funding...

by agarwalmanoj2000

Wed Mar 28, 2012 1:52 am
Forum: Critical Reasoning
Topic: weaken
Replies: 4
Views: 1277

E. Many Milville residents object to having to pay fees for recreational use of the park in the winter. Please advise, why option E is wrong? As per option E, the economic benefit will be reduced if Milville residents do not pay fees for recreational use of the park, so snowmobiling should be banned...

by agarwalmanoj2000

Mon Mar 26, 2012 3:37 pm
Forum: Critical Reasoning
Topic: Snowmobiles
Replies: 11
Views: 5270

Hi Subham, Buel and Buel’s biography provides ample raw material for questioning the myth of a colonial golden age in the eighteenth century. We need to focus on “the myth of a colonial golden age “ . It means colonial golden age was myth and not true. It is saying that colonial golden age was...

by agarwalmanoj2000

Mon Mar 19, 2012 7:57 pm
Forum: Reading Comprehension
Topic: OG RC #81
Replies: 1
Views: 1997

Happymanocha, It is OK to make REASONABLE assumptions or bring outside information for strengthening/ weakening question, so we can assume here that people who eat and exercise regularly are less prone to heart disease based on the given information. Yes in option D we can also assume that avoiding ...

by agarwalmanoj2000

Sun Mar 18, 2012 7:38 pm
Forum: Critical Reasoning
Topic: Citing the frequency
Replies: 5
Views: 1439

Your approach is OK.


It is OK to associate this question with cause and reasoning i.e A cause B can be strengthened by stating that nothing else other than A to cause B which is done by C in the above question.

HTH.

by agarwalmanoj2000

Thu Mar 15, 2012 8:07 pm
Forum: Critical Reasoning
Topic: Am I applying the correct Logic??
Replies: 3
Views: 1148